If $f(x)$ is a continously differentiable function such that $f'(x)$ is unbounded, then $f(x)$ is not...












2












$begingroup$


Suppose $f(x)$ is a continuously differentiable function on $mathbb{R}$ such that its derivative is unbounded.



Claim: $f(x)$ is not uniformly continuous.



Assume f(x) is uniformly continuous, then given $epsilon$ we have a suitable $delta$. Since $f'(x)$ is not bounded, their exists an element $c$ such that $f'(c) > k frac{epsilon}{delta}$, $k gg 0$. Since $f'(x)$ is continuous, if we choose a $delta$ neighbourhood of $c$, we would then by mean value theorem have
$$f(b) - f(a) = f'(c')(b-a), c' in Big(c- frac{delta}{2},; c + frac{delta}{2}Big).$$ or



$$ f'(c') = frac{epsilon}{delta}$$ - a contradiction since $f'(c')$ has value in around some small neighborhood of $k frac{epsilon}{delta}$.



So finally I have this conclusion:




If $f(x)$ is continuously differential function, then $f(x)$ is uniformly continuous iff $f'(x)$ is bounded?




Is my conclusion right?



New details:



I realize that that my argument $f'(c) to f'(c')$ is slippery and might not yield the desired conclusion every time, but suppose if I have a specific $f(x)$ and I can somehow show that $m leq mid f'(x) mid leq M$ for some interval $I$ where $f'(c) = kfrac{epsilon}{delta}$ and $m, M$ are suitable close enough (say $epsilon^{1000000})$, then does my argument holds?










share|cite|improve this question











$endgroup$

















    2












    $begingroup$


    Suppose $f(x)$ is a continuously differentiable function on $mathbb{R}$ such that its derivative is unbounded.



    Claim: $f(x)$ is not uniformly continuous.



    Assume f(x) is uniformly continuous, then given $epsilon$ we have a suitable $delta$. Since $f'(x)$ is not bounded, their exists an element $c$ such that $f'(c) > k frac{epsilon}{delta}$, $k gg 0$. Since $f'(x)$ is continuous, if we choose a $delta$ neighbourhood of $c$, we would then by mean value theorem have
    $$f(b) - f(a) = f'(c')(b-a), c' in Big(c- frac{delta}{2},; c + frac{delta}{2}Big).$$ or



    $$ f'(c') = frac{epsilon}{delta}$$ - a contradiction since $f'(c')$ has value in around some small neighborhood of $k frac{epsilon}{delta}$.



    So finally I have this conclusion:




    If $f(x)$ is continuously differential function, then $f(x)$ is uniformly continuous iff $f'(x)$ is bounded?




    Is my conclusion right?



    New details:



    I realize that that my argument $f'(c) to f'(c')$ is slippery and might not yield the desired conclusion every time, but suppose if I have a specific $f(x)$ and I can somehow show that $m leq mid f'(x) mid leq M$ for some interval $I$ where $f'(c) = kfrac{epsilon}{delta}$ and $m, M$ are suitable close enough (say $epsilon^{1000000})$, then does my argument holds?










    share|cite|improve this question











    $endgroup$















      2












      2








      2





      $begingroup$


      Suppose $f(x)$ is a continuously differentiable function on $mathbb{R}$ such that its derivative is unbounded.



      Claim: $f(x)$ is not uniformly continuous.



      Assume f(x) is uniformly continuous, then given $epsilon$ we have a suitable $delta$. Since $f'(x)$ is not bounded, their exists an element $c$ such that $f'(c) > k frac{epsilon}{delta}$, $k gg 0$. Since $f'(x)$ is continuous, if we choose a $delta$ neighbourhood of $c$, we would then by mean value theorem have
      $$f(b) - f(a) = f'(c')(b-a), c' in Big(c- frac{delta}{2},; c + frac{delta}{2}Big).$$ or



      $$ f'(c') = frac{epsilon}{delta}$$ - a contradiction since $f'(c')$ has value in around some small neighborhood of $k frac{epsilon}{delta}$.



      So finally I have this conclusion:




      If $f(x)$ is continuously differential function, then $f(x)$ is uniformly continuous iff $f'(x)$ is bounded?




      Is my conclusion right?



      New details:



      I realize that that my argument $f'(c) to f'(c')$ is slippery and might not yield the desired conclusion every time, but suppose if I have a specific $f(x)$ and I can somehow show that $m leq mid f'(x) mid leq M$ for some interval $I$ where $f'(c) = kfrac{epsilon}{delta}$ and $m, M$ are suitable close enough (say $epsilon^{1000000})$, then does my argument holds?










      share|cite|improve this question











      $endgroup$




      Suppose $f(x)$ is a continuously differentiable function on $mathbb{R}$ such that its derivative is unbounded.



      Claim: $f(x)$ is not uniformly continuous.



      Assume f(x) is uniformly continuous, then given $epsilon$ we have a suitable $delta$. Since $f'(x)$ is not bounded, their exists an element $c$ such that $f'(c) > k frac{epsilon}{delta}$, $k gg 0$. Since $f'(x)$ is continuous, if we choose a $delta$ neighbourhood of $c$, we would then by mean value theorem have
      $$f(b) - f(a) = f'(c')(b-a), c' in Big(c- frac{delta}{2},; c + frac{delta}{2}Big).$$ or



      $$ f'(c') = frac{epsilon}{delta}$$ - a contradiction since $f'(c')$ has value in around some small neighborhood of $k frac{epsilon}{delta}$.



      So finally I have this conclusion:




      If $f(x)$ is continuously differential function, then $f(x)$ is uniformly continuous iff $f'(x)$ is bounded?




      Is my conclusion right?



      New details:



      I realize that that my argument $f'(c) to f'(c')$ is slippery and might not yield the desired conclusion every time, but suppose if I have a specific $f(x)$ and I can somehow show that $m leq mid f'(x) mid leq M$ for some interval $I$ where $f'(c) = kfrac{epsilon}{delta}$ and $m, M$ are suitable close enough (say $epsilon^{1000000})$, then does my argument holds?







      uniform-continuity






      share|cite|improve this question















      share|cite|improve this question













      share|cite|improve this question




      share|cite|improve this question








      edited Dec 6 '18 at 12:49







      henceproved

















      asked Nov 30 '18 at 9:17









      henceprovedhenceproved

      1358




      1358






















          3 Answers
          3






          active

          oldest

          votes


















          4












          $begingroup$

          Take any unbounded continuous integrable function $g$ and define $f(x)=int_0^{x} g(y), dy$. Then $f$ is uniformly continuous but its derivative is not bounded. To construct such a function $g$ draw triangles with bases $(n-frac 1 {n^{3}}, n+frac 1 {n^{3}})$ and height $n$. Think of a function $g$ whose graph is made up of these triangles. $f$ is uniformly continuous because it is continuous and it has finite limits at $pm infty$.






          share|cite|improve this answer











          $endgroup$





















            1












            $begingroup$

            The error in OP's argument is that they fix $epsilon, delta$ beforehand for the uniformity claim, but later use the fact that the derivative $f'$ is above a certain value in an interval of length $delta$, which is not guaranteed, as Kavi explained.



            Intuitively, if the interval where $f'$ is high is very small, and becomes smaller with higher $f'$, for a given $epsilon$ the 'spikes' in $f'$ (as described by Kavi) are not able to increase $f$ by much (more than $epsilon$) except for a finite number at the beginning.






            share|cite|improve this answer









            $endgroup$





















              1












              $begingroup$

              No, you are not right. Consider for example the $C^{infty}$ function
              $$f(x)=frac{cos(e^x)}{1+x^2}.$$
              $f$ is uniformly continuous in $mathbb{R}$ because it is continuous and its limit at $pm infty$ is zero.



              On the other hand, its derivative is unbounded in $mathbb{R}$:
              $$f'(x)=-frac{e^xsin(e^x)}{1+x^2}-frac{2xcos(e^x)}{(1+x^2)^2}$$
              and $lim_{nto +infty} f'(x_n)=+infty$ where $x_n=ln(3pi/2+2pi n)$.






              share|cite|improve this answer











              $endgroup$













                Your Answer





                StackExchange.ifUsing("editor", function () {
                return StackExchange.using("mathjaxEditing", function () {
                StackExchange.MarkdownEditor.creationCallbacks.add(function (editor, postfix) {
                StackExchange.mathjaxEditing.prepareWmdForMathJax(editor, postfix, [["$", "$"], ["\\(","\\)"]]);
                });
                });
                }, "mathjax-editing");

                StackExchange.ready(function() {
                var channelOptions = {
                tags: "".split(" "),
                id: "69"
                };
                initTagRenderer("".split(" "), "".split(" "), channelOptions);

                StackExchange.using("externalEditor", function() {
                // Have to fire editor after snippets, if snippets enabled
                if (StackExchange.settings.snippets.snippetsEnabled) {
                StackExchange.using("snippets", function() {
                createEditor();
                });
                }
                else {
                createEditor();
                }
                });

                function createEditor() {
                StackExchange.prepareEditor({
                heartbeatType: 'answer',
                autoActivateHeartbeat: false,
                convertImagesToLinks: true,
                noModals: true,
                showLowRepImageUploadWarning: true,
                reputationToPostImages: 10,
                bindNavPrevention: true,
                postfix: "",
                imageUploader: {
                brandingHtml: "Powered by u003ca class="icon-imgur-white" href="https://imgur.com/"u003eu003c/au003e",
                contentPolicyHtml: "User contributions licensed under u003ca href="https://creativecommons.org/licenses/by-sa/3.0/"u003ecc by-sa 3.0 with attribution requiredu003c/au003e u003ca href="https://stackoverflow.com/legal/content-policy"u003e(content policy)u003c/au003e",
                allowUrls: true
                },
                noCode: true, onDemand: true,
                discardSelector: ".discard-answer"
                ,immediatelyShowMarkdownHelp:true
                });


                }
                });














                draft saved

                draft discarded


















                StackExchange.ready(
                function () {
                StackExchange.openid.initPostLogin('.new-post-login', 'https%3a%2f%2fmath.stackexchange.com%2fquestions%2f3019862%2fif-fx-is-a-continously-differentiable-function-such-that-fx-is-unbounde%23new-answer', 'question_page');
                }
                );

                Post as a guest















                Required, but never shown

























                3 Answers
                3






                active

                oldest

                votes








                3 Answers
                3






                active

                oldest

                votes









                active

                oldest

                votes






                active

                oldest

                votes









                4












                $begingroup$

                Take any unbounded continuous integrable function $g$ and define $f(x)=int_0^{x} g(y), dy$. Then $f$ is uniformly continuous but its derivative is not bounded. To construct such a function $g$ draw triangles with bases $(n-frac 1 {n^{3}}, n+frac 1 {n^{3}})$ and height $n$. Think of a function $g$ whose graph is made up of these triangles. $f$ is uniformly continuous because it is continuous and it has finite limits at $pm infty$.






                share|cite|improve this answer











                $endgroup$


















                  4












                  $begingroup$

                  Take any unbounded continuous integrable function $g$ and define $f(x)=int_0^{x} g(y), dy$. Then $f$ is uniformly continuous but its derivative is not bounded. To construct such a function $g$ draw triangles with bases $(n-frac 1 {n^{3}}, n+frac 1 {n^{3}})$ and height $n$. Think of a function $g$ whose graph is made up of these triangles. $f$ is uniformly continuous because it is continuous and it has finite limits at $pm infty$.






                  share|cite|improve this answer











                  $endgroup$
















                    4












                    4








                    4





                    $begingroup$

                    Take any unbounded continuous integrable function $g$ and define $f(x)=int_0^{x} g(y), dy$. Then $f$ is uniformly continuous but its derivative is not bounded. To construct such a function $g$ draw triangles with bases $(n-frac 1 {n^{3}}, n+frac 1 {n^{3}})$ and height $n$. Think of a function $g$ whose graph is made up of these triangles. $f$ is uniformly continuous because it is continuous and it has finite limits at $pm infty$.






                    share|cite|improve this answer











                    $endgroup$



                    Take any unbounded continuous integrable function $g$ and define $f(x)=int_0^{x} g(y), dy$. Then $f$ is uniformly continuous but its derivative is not bounded. To construct such a function $g$ draw triangles with bases $(n-frac 1 {n^{3}}, n+frac 1 {n^{3}})$ and height $n$. Think of a function $g$ whose graph is made up of these triangles. $f$ is uniformly continuous because it is continuous and it has finite limits at $pm infty$.







                    share|cite|improve this answer














                    share|cite|improve this answer



                    share|cite|improve this answer








                    edited Nov 30 '18 at 9:29

























                    answered Nov 30 '18 at 9:23









                    Kavi Rama MurthyKavi Rama Murthy

                    52.6k32055




                    52.6k32055























                        1












                        $begingroup$

                        The error in OP's argument is that they fix $epsilon, delta$ beforehand for the uniformity claim, but later use the fact that the derivative $f'$ is above a certain value in an interval of length $delta$, which is not guaranteed, as Kavi explained.



                        Intuitively, if the interval where $f'$ is high is very small, and becomes smaller with higher $f'$, for a given $epsilon$ the 'spikes' in $f'$ (as described by Kavi) are not able to increase $f$ by much (more than $epsilon$) except for a finite number at the beginning.






                        share|cite|improve this answer









                        $endgroup$


















                          1












                          $begingroup$

                          The error in OP's argument is that they fix $epsilon, delta$ beforehand for the uniformity claim, but later use the fact that the derivative $f'$ is above a certain value in an interval of length $delta$, which is not guaranteed, as Kavi explained.



                          Intuitively, if the interval where $f'$ is high is very small, and becomes smaller with higher $f'$, for a given $epsilon$ the 'spikes' in $f'$ (as described by Kavi) are not able to increase $f$ by much (more than $epsilon$) except for a finite number at the beginning.






                          share|cite|improve this answer









                          $endgroup$
















                            1












                            1








                            1





                            $begingroup$

                            The error in OP's argument is that they fix $epsilon, delta$ beforehand for the uniformity claim, but later use the fact that the derivative $f'$ is above a certain value in an interval of length $delta$, which is not guaranteed, as Kavi explained.



                            Intuitively, if the interval where $f'$ is high is very small, and becomes smaller with higher $f'$, for a given $epsilon$ the 'spikes' in $f'$ (as described by Kavi) are not able to increase $f$ by much (more than $epsilon$) except for a finite number at the beginning.






                            share|cite|improve this answer









                            $endgroup$



                            The error in OP's argument is that they fix $epsilon, delta$ beforehand for the uniformity claim, but later use the fact that the derivative $f'$ is above a certain value in an interval of length $delta$, which is not guaranteed, as Kavi explained.



                            Intuitively, if the interval where $f'$ is high is very small, and becomes smaller with higher $f'$, for a given $epsilon$ the 'spikes' in $f'$ (as described by Kavi) are not able to increase $f$ by much (more than $epsilon$) except for a finite number at the beginning.







                            share|cite|improve this answer












                            share|cite|improve this answer



                            share|cite|improve this answer










                            answered Nov 30 '18 at 9:38









                            IngixIngix

                            3,329146




                            3,329146























                                1












                                $begingroup$

                                No, you are not right. Consider for example the $C^{infty}$ function
                                $$f(x)=frac{cos(e^x)}{1+x^2}.$$
                                $f$ is uniformly continuous in $mathbb{R}$ because it is continuous and its limit at $pm infty$ is zero.



                                On the other hand, its derivative is unbounded in $mathbb{R}$:
                                $$f'(x)=-frac{e^xsin(e^x)}{1+x^2}-frac{2xcos(e^x)}{(1+x^2)^2}$$
                                and $lim_{nto +infty} f'(x_n)=+infty$ where $x_n=ln(3pi/2+2pi n)$.






                                share|cite|improve this answer











                                $endgroup$


















                                  1












                                  $begingroup$

                                  No, you are not right. Consider for example the $C^{infty}$ function
                                  $$f(x)=frac{cos(e^x)}{1+x^2}.$$
                                  $f$ is uniformly continuous in $mathbb{R}$ because it is continuous and its limit at $pm infty$ is zero.



                                  On the other hand, its derivative is unbounded in $mathbb{R}$:
                                  $$f'(x)=-frac{e^xsin(e^x)}{1+x^2}-frac{2xcos(e^x)}{(1+x^2)^2}$$
                                  and $lim_{nto +infty} f'(x_n)=+infty$ where $x_n=ln(3pi/2+2pi n)$.






                                  share|cite|improve this answer











                                  $endgroup$
















                                    1












                                    1








                                    1





                                    $begingroup$

                                    No, you are not right. Consider for example the $C^{infty}$ function
                                    $$f(x)=frac{cos(e^x)}{1+x^2}.$$
                                    $f$ is uniformly continuous in $mathbb{R}$ because it is continuous and its limit at $pm infty$ is zero.



                                    On the other hand, its derivative is unbounded in $mathbb{R}$:
                                    $$f'(x)=-frac{e^xsin(e^x)}{1+x^2}-frac{2xcos(e^x)}{(1+x^2)^2}$$
                                    and $lim_{nto +infty} f'(x_n)=+infty$ where $x_n=ln(3pi/2+2pi n)$.






                                    share|cite|improve this answer











                                    $endgroup$



                                    No, you are not right. Consider for example the $C^{infty}$ function
                                    $$f(x)=frac{cos(e^x)}{1+x^2}.$$
                                    $f$ is uniformly continuous in $mathbb{R}$ because it is continuous and its limit at $pm infty$ is zero.



                                    On the other hand, its derivative is unbounded in $mathbb{R}$:
                                    $$f'(x)=-frac{e^xsin(e^x)}{1+x^2}-frac{2xcos(e^x)}{(1+x^2)^2}$$
                                    and $lim_{nto +infty} f'(x_n)=+infty$ where $x_n=ln(3pi/2+2pi n)$.







                                    share|cite|improve this answer














                                    share|cite|improve this answer



                                    share|cite|improve this answer








                                    edited Nov 30 '18 at 16:10

























                                    answered Nov 30 '18 at 9:28









                                    Robert ZRobert Z

                                    94.2k1061132




                                    94.2k1061132






























                                        draft saved

                                        draft discarded




















































                                        Thanks for contributing an answer to Mathematics Stack Exchange!


                                        • Please be sure to answer the question. Provide details and share your research!

                                        But avoid



                                        • Asking for help, clarification, or responding to other answers.

                                        • Making statements based on opinion; back them up with references or personal experience.


                                        Use MathJax to format equations. MathJax reference.


                                        To learn more, see our tips on writing great answers.




                                        draft saved


                                        draft discarded














                                        StackExchange.ready(
                                        function () {
                                        StackExchange.openid.initPostLogin('.new-post-login', 'https%3a%2f%2fmath.stackexchange.com%2fquestions%2f3019862%2fif-fx-is-a-continously-differentiable-function-such-that-fx-is-unbounde%23new-answer', 'question_page');
                                        }
                                        );

                                        Post as a guest















                                        Required, but never shown





















































                                        Required, but never shown














                                        Required, but never shown












                                        Required, but never shown







                                        Required, but never shown

































                                        Required, but never shown














                                        Required, but never shown












                                        Required, but never shown







                                        Required, but never shown







                                        Popular posts from this blog

                                        Quarter-circle Tiles

                                        build a pushdown automaton that recognizes the reverse language of a given pushdown automaton?

                                        Mont Emei